Difference between revisions of "2021 Fall AMC 12A Problems/Problem 4"

(Video Solution)
m (Solution)
Line 6: Line 6:
 
<math>\textbf{(A)}\ 1 \qquad\textbf{(B)}\ 3 \qquad\textbf{(C)}\ 5 \qquad\textbf{(D)}\ 7 \qquad\textbf{(E)}\ 9</math>
 
<math>\textbf{(A)}\ 1 \qquad\textbf{(B)}\ 3 \qquad\textbf{(C)}\ 5 \qquad\textbf{(D)}\ 7 \qquad\textbf{(E)}\ 9</math>
  
== Solution ==
+
== Solution 1==
 
First, modulo <math>2</math> or <math>5</math>, <math>\underline{20210A} \equiv A</math>.
 
First, modulo <math>2</math> or <math>5</math>, <math>\underline{20210A} \equiv A</math>.
 
Hence, <math>A \neq 0, 2, 4, 5, 6, 8</math>.
 
Hence, <math>A \neq 0, 2, 4, 5, 6, 8</math>.
Line 18: Line 18:
 
Therefore, the answer is <math>\boxed{\textbf{(E) }9}</math>.
 
Therefore, the answer is <math>\boxed{\textbf{(E) }9}</math>.
  
~NH14
+
~NH14 ~Steven Chen (www.professorchenedu.com)
 
 
~Steven Chen (www.professorchenedu.com)
 
  
 
==Solution 2==
 
==Solution 2==

Revision as of 02:28, 18 August 2022

The following problem is from both the 2021 Fall AMC 10A #5 and 2021 Fall AMC 12A #4, so both problems redirect to this page.

Problem

The six-digit number $\underline{2}\,\underline{0}\,\underline{2}\,\underline{1}\,\underline{0}\,\underline{A}$ is prime for only one digit $A.$ What is $A?$

$\textbf{(A)}\ 1 \qquad\textbf{(B)}\ 3 \qquad\textbf{(C)}\ 5 \qquad\textbf{(D)}\ 7 \qquad\textbf{(E)}\ 9$

Solution 1

First, modulo $2$ or $5$, $\underline{20210A} \equiv A$. Hence, $A \neq 0, 2, 4, 5, 6, 8$.

Second modulo $3$, $\underline{20210A} \equiv 2 + 0 + 2 + 1 + 0 + A \equiv 5 + A$. Hence, $A \neq 1, 4, 7$.

Third, modulo $11$, $\underline{20210A} \equiv A + 1 + 0 - 0 - 2 - 2 \equiv A - 3$. Hence, $A \neq 3$.

Therefore, the answer is $\boxed{\textbf{(E) }9}$.

~NH14 ~Steven Chen (www.professorchenedu.com)

Solution 2

$202100 \implies$ divisible by $2$.

$202101 \implies$ divisible by $3$.

$202102 \implies$ divisible by $2$.

$202103 \implies$ divisible by $11$.

$202104 \implies$ divisible by $2$.

$202105 \implies$ divisible by $5$.

$202106 \implies$ divisible by $2$.

$202107 \implies$ divisible by $3$.

$202108 \implies$ divisible by $2$.

This leaves only $A=\boxed{\textbf{(E) }9}$.

~wamofan

Video Solution 1

https://youtu.be/7_Dg9b2hQ5U

~Education, the Study of Everything



Video Solution

https://youtu.be/jxnTkY3eb5Y

~savannahsolver

https://youtu.be/AgzDyKlmNAo

~Charles3829

Video Solution by TheBeautyofMath

for AMC 10: https://youtu.be/o98vGHAUYjM?t=623

for AMC 12: https://youtu.be/jY-17W6dA3c?t=392

~IceMatrix

Video Solution

https://youtu.be/7TnFYSJ8i14

~Lucas

See Also

2021 Fall AMC 12A (ProblemsAnswer KeyResources)
Preceded by
Problem 3
Followed by
Problem 5
1 2 3 4 5 6 7 8 9 10 11 12 13 14 15 16 17 18 19 20 21 22 23 24 25
All AMC 12 Problems and Solutions
2021 Fall AMC 10A (ProblemsAnswer KeyResources)
Preceded by
Problem 4
Followed by
Problem 6
1 2 3 4 5 6 7 8 9 10 11 12 13 14 15 16 17 18 19 20 21 22 23 24 25
All AMC 10 Problems and Solutions

The problems on this page are copyrighted by the Mathematical Association of America's American Mathematics Competitions. AMC logo.png